512 Quizzes

अब Quizwiz के साथ अपने होमवर्क और परीक्षाओं को एस करें!

This year, Devin and Lana withdraw $25,000 from their variable annuity to pay some unexpected bills. Assuming the annuity was issued in 2010, and the $125,000 balance in the annuity includes $50,000 in earnings, what is the amount of the $25,000 withdrawal that is subject to income tax? A) $12,500 B) $25,000 C) $0 D) $5,000 Explanation The answer is $25,000. Because the annuity was issued after August 13, 1982, any withdrawals are taxable, to the extent there are earnings in the contract (LIFO rule). Therefore, the entire $25,000 withdrawal is subject to income tax.

B) $25,000

Which of the following is a counted asset for determining Medicaid eligibility? A) Life insurance with a face value under $1,500 B) Checking accounts C) Primary residence D) One motor vehicle Explanation Checking accounts are counted assets for determining eligibility for Medicaid. The other three assets are not counted.

B) Checking accounts

Which of the following statements regarding the Health Insurance Portability and Accountability Act of 1996 (HIPAA) is CORRECT? A) HIPAA defined a pre-existing condition as a medical condition that was treated or diagnosed within three months before enrolling in a new group health plan. B) HIPAA defined a pre-existing condition as a medical condition that was treated or diagnosed within six months before enrolling in a new group health plan. C) HIPAA defined a pre-existing condition as a medical condition that was treated or diagnosed within nine months before enrolling in a new group health plan. D) HIPAA defined a pre-existing condition as a medical condition that was treated or diagnosed within 12 months before enrolling in a new group health plan. Explanation The answer is HIPAA defined a pre-existing condition as a medical condition that was treated or diagnosed within six months before enrolling in a new group health plan.

B) HIPAA defined a pre-existing condition as a medical condition that was treated or diagnosed within six months before enrolling in a new group health plan.

Which of the following is an advantage of single-premium immediate annuities (SPIAs)? Investment risk is transferred to the insurer. They protect the principal from creditors. Most benefits increase with inflation. They ensure a lifetime of income, without fear of outliving the principal. A) I and IV B) I, II, and IV C) I and II D) II and III Explanation The answer is I, II, and IV. Statement III is incorrect. Most SPIA benefits are fixed and will not increase with inflation unless the owner has included a cost-of-living adjustment rider.

B) I, II, and IV

Devin and Lana are thinking about selecting a settlement option for their variable annuity. Their objective is to have the annuity provide income until both of them are deceased. Which of the following settlement options will best meet their needs? A) Life annuity with period certain B) Joint and survivor annuity C) Installment refund annuity D) Single (straight) life annuity Explanation The joint and survivor annuity will provide the couple with annuity income until both of them are deceased. For a given purchase price, a single (straight) life annuity generally provides the highest monthly payment amount because the annuity provides no payments beyond the annuitant's life.

B) Joint and survivor annuity

Tamu Enterprises, Inc., would like to set up an equity-based compensation plan that has the following features: Shares that are a separate class of common stock Shares that are usually convertible into common shares Shares that allow employees to purchase company stock below fair market value Shares with rights that are subordinate to regular common stock shares Which of the following would be the best choice for the company? A) Incentive stock options B) Junior class shares C) Restricted stock D) Stock appreciation rights Explanation Junior class shares are a separate class of common stock. The voting, liquidation, and dividend rights of junior class shares are subordinate to regular common stock shares. Junior class shares are generally convertible into common shares of stock upon specified requirements, such as achieving certain years of service or performance goals. Conversion can be automatic or at the employee's discretion.

B) Junior class shares

Which of the following is a pitfall to avoid when selecting an appropriate type of life insurance? A) Establishing client goals B) Recommending the purchase of term insurance because the client is young C) Comparing the costs of available types of life insurance D) Taking advantage of breakpoints Explanation The answer is recommending the purchase of term insurance because the client is young. Youth, by itself, is not the determining factor in deciding upon the type of policy to be purchased. Specifically, a strong argument can be made that premiums used to purchase term insurance could have been earning a cash value in a more permanent type of insurance. The type of insurance to be purchased should be determined by the factors comprising the client profile. Insurance companies often have breakpoints with regard to pricing. For instance, it is likely a $500,000 policy will cost less than a $490,000 policy.

B) Recommending the purchase of term insurance because the client is young

Allison is a key employee for JEM Company. She is offered group term life insurance for two times her base salary of $100,000, while rank-and-file employees are only offered one times their base salary. Based on this information, how much of the premium on her group term life insurance coverage must be included in her taxable compensation for the year? A) The premium for $50,000 of coverage B) The premium on $200,000 of coverage C) $0 D) The premium for $150,000 of coverage Explanation The answer is the premiums on $200,000. Because she is a key employee, and the group term life insurance plan discriminated on her behalf, the entire amount of the premium paid by JEM attributable to the death benefit would be subject to ordinary income tax.

B) The premium on $200,000 of coverage

Group major medical benefits usually begin after the insured has incurred an initial annual amount of out-of-pocket expense called A) a coinsurance limit. B) a deductible. C) a stop-loss limit. D) a maximum out-of-pocket limit. Explanation The answer is a deductible. The deductible requires insureds to pay a certain amount of their medical expenses each calendar year before coverage begins.

B) a deductible.

All of the following are considered activities of daily living (ADLs) under the Health Insurance Portability and Accountability Act (HIPAA) except A) eating. B) talking. C) maintaining continence. D) transferring from bed to chair. Explanation The answer is talking. Talking is not an ADL. The other activities are all ADLs under HIPAA.

B) talking.

The sharing of expenses between the insured and the insurer in a group health insurance plan is an example of A) the maximum out-of-pocket (MOOP). B) the coinsurance. C) the maximum benefit. D) the deductible. Explanation The sharing of expenses between the insured and the insurer in a group health insurance plan is an example of coinsurance. A common split between insurer and insured is 80/20.

B) the coinsurance.

For a group life insurance plan to not be considered discriminatory, A) employees must receive the same dollar amount of coverage. B) the same formula for coverage amounts must be applied to all employees. C) benefits would have to be the same for key and nonkey employees. D) it must pass either the ratio percentage test or the average benefits test. Explanation The answer is the same formula for coverage amounts must be applied to all employees. Uniformity of benefit amounts does not mean that all employees must receive the same dollar amount of coverage—only that the same formula for coverage amounts must be applied to all employees. Thus, all benefits available to key employees must be available to all participants using a uniform formula.

B) the same formula for coverage amounts must be applied to all employees.

All of the following statements regarding unemployment insurance are correct except A) the employer pays a tax to fund the unemployment benefit to the out-of-work employee. B) unemployment compensation is tax free to the recipient. C) typically, unemployment benefits are paid out for 26 weeks. D) benefits are determined by previous wage levels. Explanation The answer is unemployment compensation is tax free to the recipient. Unemployment compensation is fully taxable to the recipient.

B) unemployment compensation is tax free to the recipient.

All of the following statements regarding whole life insurance are correct except A) whole life is considered permanent life insurance. B) whole life insurance pays benefits only if the insured dies during a specified period of years. C) the protection afforded by the whole life insurance contract is permanent. The term never expires, and the policy never has to be renewed or converted. D) whole life insurance policies issued on a participating basis may return part of the premium in the form of policyowner dividends. Explanation Whole life insurance provides for the payment of the policy's face amount upon the death of the insured, regardless of when death occurs.

B) whole life insurance pays benefits only if the insured dies during a specified period of years.

Which of the following statements regarding the principles of risk and insurance are CORRECT? Risk is a condition in which there is a possibility of an adverse result from the expected desired outcome. A hazard is the cause of a financial loss and is the actual event for which the individual purchases insurance. A peril is a condition that increases the probability that a financial loss will occur. Pure risk involves only the chance of loss or no loss. A) I and II B) I and IV C) I, II, and III D) I, II, III, and IV Explanation Risk is the condition in which there is a possibility of an adverse result from the expected desired outcome. A peril is the cause of a financial loss, and a hazard is a condition that increases the chance of financial loss. Pure risk is the risk that only involves the chance of loss or no loss.

B) I and IV

Under HIPAA, a chronically ill person is unable to perform _____ activities of daily living (ADLs) for a period of at least _____ days. A) 3, 60 B) 3, 30 C) 2, 90 D) 2, 60 Explanation The answer is 2, 90. Under a qualified long-term care policy, a chronically ill person is expected to be unable to perform, without substantial assistance from another person, two ADLs for 90 days.

C) 2, 90

Which of the following is a common property coverage that can be included in a businessowners policy (BOP)? A) Workers' compensation B) Professional liability C) Inland marine D) General liability Explanation The answer is inland marine. Inland marine coverage is a property coverage, while the other answer choices provide liability coverage.

C) Inland marine

Which of the following dividend options allows for acquiring additional insurance with no underwriting? A) Reduced premium B) Cash C) One-year term D) Accumulate at interest Explanation The answer is one-year term. With the one-year term dividend option, the dividend is used to purchase additional term death benefit protection with no underwriting required. The paid-up additions and one-year term dividend options both allow the policyowner to acquire more death benefit at no additional cost, other than using the dividend instead of taking it as cash.

C) One-year term

Which of the following states that the insured should not profit from a covered loss, but should be restored to approximately the same financial position before the occurrence of the loss? A) Collateral source rule B) Principle of insurable interest C) Principle of indemnity D) Rule of adhesion Explanation The answer is principle of indemnity. The principle of indemnity has two purposes: to prevent the insured from profiting from insurance claims and to reduce moral hazards.

C) Principle of indemnity

In which of the following situations would it NOT be appropriate for the employer company to implement an equity-based compensation plan? A) Scott Technology wants to provide a deferred compensation benefit to its executives, but it has many employees, and the cost to implement this benefit for all would be excessive. B) Eagle Sporting Goods wants to provide certain key employees with tax-deferred compensation under terms or conditions different to those applicable to the rank-and-file employees. C) Steiner Industries wants to receive a deduction at the time of contribution to the plan. D) Domingo Foods wants to provide additional deferred compensation benefits to an executive who is already receiving maximum contributions under the company's existing retirement plan. Explanation If Steiner Industries wants to receive a deduction at the time of contribution, the company should consider a qualified plan, not an equity-based compensation plan. Nonqualified plans do not permit the employer to take a deduction for plan contributions until the employee reports income from the plan, which is often at retirement.

C) Steiner Industries wants to receive a deduction at the time of contribution to the plan.

All of the following are mandatory benefits under Medicaid except A) inpatient hospital services. B) physician services. C) chiropractic services. D) outpatient hospital services. Explanation Chiropractic services are optional, not mandatory, Medicaid benefits.

C) chiropractic services.

Justin's employer grants him 1,000 shares of restricted stock worth $10 per share at the time of the grant. Justin pays nothing for the grant. The terms of the restriction require him to remain employed by his company for an additional five years. When he meets the five-year requirement, the stock is trading for $25 per share. What is the amount Justin must report as compensation (W-2) income for the year in which he meets the five-year requirement? A) $0 B) $12,500 C) $10,000 D) $25,000 Explanation Justin must report the value of the stock as compensation income in the year in which the stock is no longer subject to a substantial risk of forfeiture. Therefore, he must report $25,000 ($25 × 1,000 shares) of income for the current year.

D) $25,000

Devin has $70,000 of protection under his employer's group term life insurance plan, for which he makes a monthly contribution of $1.50. Lana is the named beneficiary. The actual cost to the employer for his protection is $0.20 per month per $1,000 of insurance, and the uniform premium for group term insurance under the Internal Revenue Code is $0.09 per month per $1,000 of insurance. Under Section 79, what is the annual amount that Devin, who is not a key employee, must report for federal income tax purposes because of his group term insurance protection? A) $21.60 B) $30.00 C) $57.60 D) $3.60 Explanation The answer is $3.60. The amount is calculated by multiplying $0.09 per month by 20 ($20,000 of coverage in excess of $50,000) and then multiplying by 12 to arrive at the annual amount (0.09 × 20 × 12 = $21.60). Then, subtract the amount paid by the employee ($1.50 per month × 12 months = $18.00 per year); $21.60 − $18.00 = $3.60.

D) $3.60

When evaluating medical insurance coverage, which of the following factors should be considered? Whether the doctors and hospitals the client wishes to use are in network. Total projected out-of-pocket costs including deductibles, copays, and maximum out-of-pocket expense limits. A) I only B) II only C) Neither I nor II D) Both I and II Explanation Both of these factors should be considered.

D) Both I and II

Which of the following is a common characteristic of group health insurance utilizing a preferred provider network? Participating providers are paid on a fee-for-service basis, as their services are used by covered employees. Covered employees have financial incentives to receive treatment within the network. A) I only B) II only C) Neither I nor II D) Both I and II Explanation Both statements I and II are correct.

D) Both I and II

For life insurance, an insurable interest must exist when the policy is issued. when the insured dies. A) Both I and II B) Neither I nor II C) II only D) I only Explanation For life insurance to be underwritten, an insurable interest need only exist when the policy is issued

D) I only

In evaluating the replacement of life insurance, which of the following does NOT need to be considered? Spouse's health status New front-end load costs Income tax consequences A) II and III B) I and III C) I and II D) I only Explanation The answer is I only. The client's health status would impact replacement, but the spouse's health would have no impact. All other factors should be considered.

D) I only

All of the following statements regarding partial and residual benefit payment provisions are correct EXCEPT A) they exist to encourage people to return to work. B) they are invoked after periods of total disability. C) they may be part of a base policy or added as riders. D) they guarantee benefits even after claimants are earning 100% of their predisability income. Explanation These provisions are structured to correspond to percentages of lost income. Barring fraud, it is highly unlikely that an individual could receive benefits after achieving 100% of their predisability income.

D) they guarantee benefits even after claimants are earning 100% of their predisability income.

Which of the following is the correct period of continuation coverage for an employee who voluntarily terminates employment under COBRA? A) 36 months B) 18 months C) 60 months D) 0 months Explanation The answer is 18 months. Under COBRA, the required period of continuation coverage following termination of employment, whether voluntary or involuntary, is 18 months.

B) 18 months

Which of the following is an insurance producer who represents the applicant for insurance? A) A producing general agent B) A career agent C) An independent agent D) A broker Explanation The answer is a broker. Brokers represent potential insureds. Agents represent insurance companies.

D) A broker

Robin is granted 1,500 shares of restricted stock from her employer when the stock is trading at a fair market value of $25 per share. She is anticipating significant appreciation and wishes to minimize her future tax burden. As a result, she makes a Section 83(b) election. Assuming she is in the 35% marginal income tax bracket, how much income tax will be due on this transaction in the year of election? A) $13,125 B) $50,625 C) $37,500 D) $24,375 Explanation She will owe $13,125 [(1,500 shares × $25 per share) × 35% tax bracket] of income tax on this transaction.

A) $13,125

Monica is an employee of ABC, Inc., which provides group term life insurance (GTLI) to each employee. Monica has coverage in the amount of $160,000. The monthly cost for this GTLI coverage is $0.38 per $1,000. Monica makes annual payments of $200 toward the cost of the insurance. What amount must be reported as income on her W-2 for the cost of her GTLI? A) $301.60 B) $501.60 C) $41.80 D) $0 Explanation The answer is $301.60. The amount that must be reported as taxable income on Monica's W-2 is $301.60, calculated as the following: GTLI coverage$160,000Less− 50,000Excess coverage$110,000Times cost per $1,000× 0.38Monthly cost$ 41.80Times 12 months× 12Annual cost$ 501.60Monica's payment− 200.00Taxable income$ 301.60

A) $301.60

Which of the following individuals are eligible for coverage under Medicare? A person who is at least 65 years old A person who has been receiving Social Security disability benefits for at least 24 months A person who is receiving kidney dialysis treatments and is in end-stage renal failure A) I only B) I, II, and III C) I and II D) I and III Explanation The answer is I, II, and III. All of these individuals are eligible for Medicare coverage.

B) I, II, and III

Zhang has a personal auto policy, and the liability coverage is displayed as 100/300/50. How much property damage liability coverage does Zhang have? A) $50,000 B) $300,000 C) $350,000 D) $100,000 Explanation The answer is $50,000. When liability coverage is displayed in the format shown, the first number represents the thousands of dollars of bodily injury liability coverage per person. The second number is the amount in thousands of bodily injury liability coverage per accident. The third number represents the amount in thousands of property damage liability coverage. In this case, Zhang has $100,000 in bodily injury liability coverage per person, $300,000 in bodily injury liability coverage per accident, and $50,000 in property damage coverage per accident.

A) $50,000

Dana owns a building that she purchased for $600,000. Its current replacement cost is $1.5 million. The building is covered up to $900,000 for fire-related perils by ZRP Insurance Company, with an 80% coinsurance provision and a $2,000 deductible. Last week, a fire broke out in the building, causing $800,000 in covered damage. What amount will ZRP Insurance Company pay for this loss? A) $598,000 B) $800,000 C) $798,000 D) $478,000 Explanation The answer is $598,000. It is computed as follows: amount of insurance required = $1,500,000 × .8 = $1,200,000 percentage of current insurance vs. insurance required = $900,000 ÷ $1,200,000 = .75 (percentage of insurance × loss incurred) − deductible = (.75 × $800,000) − $2,000 = $598,000

A) $598,000

Donald has an HO-3 homeowners policy. His policy limit under Coverage E is $300,000. He is entertaining friends at his house one evening for a barbeque. Donald carelessly tries to start the grill with gasoline instead of lighter fluid, and one of his guests is severely burned as a result. The fire also causes $5,000 in damage to the guest's car. The burned guest sues Donald for negligence. The cost of Donald's legal defense is $20,000. At trial, the court orders Donald to pay the guest $60,000 for his medical expenses and $5,000 for the damage to his car. What amount will Coverage E of Donald's HO-3 policy pay as a result of this occurrence? A) $85,000 B) $5,000 C) $60,000 D) $80,000 Explanation The answer is $85,000. Coverage E protects the insured against claims arising out of both bodily injury and property damage. This coverage also pays the costs of defending the insured against a covered claim. Donald's policy limit of $300,000 is sufficient to cover the $5,000 in physical damage, the $60,000 in bodily injury, and the $20,000 for Donald's legal defense.

A) $85,000

Which of the following definitions of a term used in insurance contract law is CORRECT? A) Adhesion: if you wrote the contract, you are bound by any ambiguities you created. B) Personal: the dollars that exchange hands are of unequal amounts. C) Indemnity: the insurance company pays on the condition of a loss. D) Aleatory: only one party can enforce the contract. Explanation A contract of adhesion maintains that, if you wrote the contract, you are bound by any ambiguities you created.

A) Adhesion: if you wrote the contract, you are bound by any ambiguities you created.

Which of the following describes the tendency of persons with a higher-than-average chance of loss to seek insurance at standard rates, which, if not controlled by underwriting, results in higher-than-expected loss levels? A) Adverse selection B) Moral hazard C) Physical hazard D) Contributory negligence Explanation This describes adverse selection. Physical hazard is a physical condition that increases the chance of loss. Under the doctrine of contributory negligence, if any negligence on the part of the injured party contributes to the injury, it absolves the other party of liability. A moral hazard is a result of individuals being unethical or misrepresenting themselves to obtain insurance or to induce the payment of claims.

A) Adverse selection

Le'Veon wants to purchase a life insurance policy on his own life. He is concerned that the policy may lapse if he inadvertently forgets to pay the premiums. Le'Veon's family has a history of medical issues, and he is concerned that he may become uninsurable in the future. Which of the following policy provisions would best address Le'Veon's concerns? A) Automatic premium loan provision B) Reinstatement provision C) Nonforfeiture provision D) Contestability provision Explanation The answer is automatic premium loan provision. The automatic premium loan provision provides that the premium will automatically be charged against the policy cash value if it is not paid by the due date. A reinstatement provision allows a policyholder to reinstate a policy after it lapses, but only if the insured can prove insurability. A nonforfeiture provision specifies what will happen to the cash value if the policyowner discontinues premium payments, and an incontestability provision prevents the insurer from challenging the validity of a policy after it has been in force for a specific period.

A) Automatic premium loan provision

If Devin's nonqualified deferred compensation plan is informally funded, which of the following statements regarding his plan is CORRECT? I. The assets funding the plan are owned by Magnum Corporation and are subject to the claims of its general creditors. II. Any earnings generated on the assets funding the plan are taxed to Magnum Corporation. A) Both I and II B) I only C) II only D) Neither I nor II Explanation The answer is both I and II. These statements are correct if Devin's plan is informally funded.

A) Both I and II

Medicare Part D is intended to provide protection for seniors who have considerable prescription drug costs and covers: generic drugs brand-name drugs A) Both I and II B) Neither I nor II C) I only D) II only Explanation The answer is both I and II. Medicare Part D covers both generic and brand-name drugs.

A) Both I and II

Which of the following covered losses are included in a personal automobile policy? Injuries to the insured or family members Damage to the insured's vehicle A) Both I and II B) Neither I nor II C) II only D) I only Explanation The answer is both I and II. Another covered loss is legal liability for injuries and damages to other persons.

A) Both I and II

Which of the following statements concerning a personal liability umbrella policy (PLUP) is CORRECT? A PLUP is designed primarily to provide liability coverage for catastrophic legal claims or judgments. A PLUP requires the policyowner to carry certain underlying liability coverages of specified minimum amounts. A) Both I and II B) II only C) Neither I nor II D) I only Explanation The answer is both I and II. A PLUP is structured to provide a layer of liability insurance coverage for catastrophic legal claims or judgments, on top of the individual's homeowners and automobile insurance policies. To be eligible for umbrella coverage, an insurer may require the insured to maintain minimum amounts of underlying liability coverage.

A) Both I and II

Which of the following statements concerning short-term and long-term disability income plans is CORRECT? Short-term disability income contracts commonly contain an elimination period, which is the length of time a covered employee must be disabled before becoming eligible for benefits. Long-term disability policies may include coverage for both sickness and accidents. A) Both I and II B) Neither I nor II C) II only D) I only Explanation Both statements regarding short-term and long-term disability are correct.

A) Both I and II

Which of the following statements concerning variable universal life (VUL) insurance is CORRECT? VUL insurance incorporates all of the premium flexibility features of the universal life policy with the policyowner-directed investment aspects of variable life insurance. An agent selling a VUL policy must have both a state insurance license and a securities license. A) Both I and II B) Neither I nor II C) II only D) I only Explanation The answer is both I and II. A VUL insurance policy combines the policyowner investment direction element of variable life insurance with the flexible premium, cash value, and death benefit elements of universal life. Accordingly, the attributes of a VUL policy include an increasing or decreasing death benefit and a flexible premium payment schedule. Agents selling VUL policies must have a state insurance license and a securities license.

A) Both I and II

Which of the following statements pertaining to prepaid legal services offered by an employer is CORRECT? Prepaid legal services are a fringe benefit that makes legal services available to employees. Examples of benefits that may be included in the plan are bankruptcy assistance, adoption assistance, divorce legal fees assistance, and the preparation of estate planning documents. A) Both I and II B) I only C) Neither I nor II D) II only Explanation Both statements I and II are correct.

A) Both I and II

Which of the following statements regarding the income tax treatment of policy loans from modified endowment contracts (MECs) is CORRECT? They are subject to last-in, first-out (LIFO) tax treatment. They may be subject to a 10% income tax penalty if the policy owner is younger than 59½ years. A) Both I and II B) II only C) Neither I nor II D) I only Explanation The answer is both I and II. Policy loans from a MEC may be subject to a 10% income tax penalty if the policy owner is younger than 59½ years. Loans and withdrawals are also subject to LIFO tax treatment.

A) Both I and II

Because of a slowdown in business activity, Sarah has voluntarily agreed to reduce her hours and become a part-time employee. Before the change, Sarah and her spouse were covered under her employer's group health plan. Which of the following statements regarding Sarah's COBRA rights is CORRECT? A) COBRA allows for continuation coverage of 18 months in this situation. B) COBRA allows for continuation coverage of 29 months in this situation. C) COBRA allows for continuation coverage of 36 months in this situation. D) Because her change is voluntary, COBRA rules do not apply. Explanation The answer is COBRA allows for continuation coverage of 18 months in this situation. COBRA rules apply even though Sarah's move to part-time status was voluntary. She is permitted a continuation coverage period of up to 18 months.

A) COBRA allows for continuation coverage of 18 months in this situation.

The insurance provision that states that the insured will split the costs of a loss to some degree, usually on a percentage basis, describes which of the following? A) Coinsurance B) A deductible C) Subrogation D) Other insurance Explanation The answer is coinsurance. The question defines coinsurance, which is a splitting of costs. A deductible is what the insured must pay before the insurance company pays anything. It is possible to have a loss that is valued at an amount below the deductible, and nothing would be owed by the insurance company. It is not possible for the insurance company and the insured to not split costs when coinsurance applies.

A) Coinsurance

An insurance contract requires an exchange of value to be considered a legally binding document. What is the term used to describe this requirement? A) Consideration B) Legal capacity C) Legal object D) Legal form Explanation The answer is consideration. Consideration occurs when there is an exchange of value. The insurance contract requires an exchange of value to be legally binding. For most insurance, the promise to pay the premium is usually sufficient consideration to make the contract binding; however, for life insurance, the premium must be paid before the insurance contract is binding for the insurer. The submission of a completed insurance application (offer) plus the payment of the first premium (consideration) to the insurance company will generally create a binding contract if the application would pass standard underwriting requirements.

A) Consideration

Under which tax doctrine would the value of the benefit be includible as taxable income if an individual receives any economic or financial benefit or property as compensation for services? A) Economic benefit doctrine B) Substantial risk of forfeiture C) Constructive receipt doctrine D) Assignment of income doctrine Explanation Under the economic income doctrine, if an individual receives any economic or financial benefit, or property as compensation for services, the value of the benefit is includible as taxable income. Income is assumed to be received even if the employee does not have actual or constructive receipt.

A) Economic benefit doctrine

Which of the following statements best describes a characteristic of group life insurance? A) Group life insurance coverage is usually based on a percentage (or multiple) of compensation. B) The amount of coverage is usually a flat amount for each participant to avoid having the plan be discriminatory. C) Group life insurance coverage is usually based on a formula that takes into account years of service and compensation. D) Group life insurance coverage is usually based upon years of service. Explanation The answer is group life insurance coverage is usually based on a percentage (or multiple) of compensation. Group life insurance coverage usually is based on a percentage (or multiple) of compensation, thus reflecting the value of the employee to the organization and keeping pace with cost-of-living increases.

A) Group life insurance coverage is usually based on a percentage (or multiple) of compensation.

Using a seatbelt when driving, wearing a helmet when riding a bicycle, and installing a home security system are all examples of which risk management method? A) Risk reduction B) Risk retention C) Risk avoidance D) Risk transfer Explanation The answer is risk reduction. These are all examples of risk reduction. Risk avoidance would not involve avoiding driving or riding a bicycle at all. Risk retention would be choosing not to use a seatbelt or wear a helmet, or not installing a security system and taking one's chances. Risk transfer often involves insurance.

A) Risk reduction

Which statement represents the main difference between traditional health insurance arrangements and health maintenance organizations (HMOs)? A) HMOs provide both the health care service and the health care financing, but traditional health care insurance companies provide only the financing. B) HMOs provide both the health care service and the health care financing, but traditional health care insurance companies provide only the service. C) Traditional health insurance companies provide both the health care service and the health care financing, but HMOs provide only the health care financing. D) Traditional health care insurance companies provide both the health care service and the health care financing, but HMOs provide only the health care service. Explanation HMOs provide both the health care service and the health care financing, but traditional health care insurance companies provide only the financing.

A) HMOs provide both the health care service and the health care financing, but traditional health care insurance companies provide only the financing.

Which of the following statements describe circumstances that result in taxation to the employee of medical expense reimbursements from a group medical insurance policy? Medical expense reimbursements are taxable to the employee to the extent they exceed the expenses incurred. Medical expense reimbursements are taxable to the employee if the employer paid the premiums for coverage. If the employee deducted the medical expenses in a prior year, reimbursement for those expenses in a subsequent year is subject to tax. Medical expense reimbursements from an employer's nondiscriminatory self-funded plan are taxable to the employee. A) I and III B) II, III, and IV C) I, II, and III D) I and IV Explanation The answer is I and III. Both are true statements. Even if the employer pays the premiums, the benefits are not taxable to the employee. Medical expense reimbursements are not taxable to the employee unless they exceed the expenses incurred, or the employee deducted the expense.

A) I and III

Wanda is considering replacement of her life insurance policy with a new policy issued by another company. Which of the following statements regarding policy replacement is CORRECT? The new policy will have high initial costs relative to the current policy's costs. The new policy's incontestable clause will be waived. A) I only B) Neither I nor II C) II only D) Both I and II Explanation The answer is I only. The majority of a life insurance policy's costs are included in the early years of the policy. At this time, acquisition costs are high. As policies age, the expenses decrease. A new policy will have a new contestable clause, initial expenses, and the potential for a substandard rating, as it will be underwritten at the insured's current age and health.

A) I only

​One of the steps in the risk management process is to "gather pertinent data to determine risk​ ​exposures." Potential sources of necessary information include which of the following areas? Property Liability Personal injury Personal illness A) I, II, III, and IV B) I and IV C) I, II, and III D) II, III, and IV Explanation Sources of information regarding property, liability, personal injury, and personal illness are all gathered during this step.

A) I, II, III, and IV

Corporate-owned life insurance is a very popular funding vehicle for nonqualified plans because the cash value buildup is not currently taxed. the insurance policy may be purchased on the executive's life and still be owned by the employer to fund the promised benefit. these policies provide the funds to pay the benefit in the event of the executive's death before retirement. A) I, II, and III B) I and II C) I and III D) II and III Explanation The answer is I, II, and III. All of these statements are correct.

A) I, II, and III

Which of the following clients would be a good candidate for term life insurance? Tina, 21, who wants life insurance for only 20 years until her mortgage is paid off and who does not want to accumulate a cash value Ralph, 40, who wants life insurance only until he and his wife reach age 66 and begin receiving Social Security benefits. He wants the maximum face amount he can obtain for the amount of premium he can afford to pay. Sam, 35, who wants current life insurance protection at a minimal cost. He may or may not want to renew his policy from year to year, and he does not mind if his premiums increase in the future. A) I, II, and III B) II and III C) I only D) I and II Explanation The answer is I, II, and III. All of these clients are good candidates for term life insurance because they have a temporary need for life insurance protection. In addition, term life insurance does not accumulate cash value (making it appropriate for Tina) and provides the maximum face amount of coverage for a given amount of premium (making it appropriate for Ralph). Although premiums typically increase when term life insurance is renewed, this is not a disadvantage for Sam.

A) I, II, and III

Which of the following expenses is typically payable from a flexible spending account (FSA)? Medical Dental Dependent care Individual life insurance premiums A) I, II, and III B) II, III, and IV C) I only D) I, II, III, and IV Explanation The answer is I, II, and III. FSAs are usually used only for medical, dental, and dependent care expenses. Individual life insurance premiums are not covered because the IRS does not consider them a qualified benefit.

A) I, II, and III

Which of the following are true for a businessowners policy (BOP)? A BOP includes property and liability coverage. There are four parts to a BOP. BOPs are a specific, standard package of coverage. BOPs can be customized to a specific business. A) I, II, and IV B) I and III C) II and IV D) I, II, and III Explanation The answer is I, II, and IV. A BOP includes both property and liability coverage and consists of four parts: common policy conditions, property coverage, causes of loss and exclusions, and liability coverage). BOPs are customizable so that the coverage a business needs can be added to the basic framework of the policy.

A) I, II, and IV

Which of the following are principles of the workers' compensation laws? In exchange for benefits, an employee gives up the right to sue the employer, except in extreme cases. The costs for workers' compensation benefits are funded through payroll taxes, and the employee is expected to contribute. The injured employee is not required to prove negligence on the part of the employer. The benefits payable under workers' compensation are periodic payments. A) I, III, and IV B) II and III C) II, III and IV D) I and IV Explanation The answer is I, III, and IV. Statement II is incorrect. Workers' compensation is funded through premiums paid solely by the employer.

A) I, III, and IV

Darwin, a key employee of International Container Inc., is covered by the company's noncontributory group term life insurance coverage in the amount of $80,000. He has designated his spouse as a beneficiary of the policy. Which of the following statements apply to this coverage? The employer can deduct the premiums on the first $50,000 of coverage only. Darwin must pay tax on the cost of coverage above $50,000. If the plan is discriminatory, Darwin must pay tax on the cost of the entire $80,000 coverage. Darwin's taxable income will increase by $30,000—the amount of coverage in excess of $50,000. A) II and III B) II, III, and IV C) I and II D) I and III Explanation The answer is II and III. The employer can deduct the premiums on the entire $80,000 of coverage. Darwin's taxable income will increase only by the difference in the premium between $80,000 and $50,000 of coverage.

A) II and III

Question #10 of 30 Question ID: 1329368 Which of the following statements regarding Medicare Part B are CORRECT? Coverage is mandatory for anyone enrolled in Part A. Insureds must pay a monthly premium. Insureds must satisfy an annual deductible. A) II and III B) I, II, and III C) III only D) I and II Explanation Statement I is incorrect. Enrollment in Part B is voluntary, although enrollment is on an opt-out, not an opt-in, basis.

A) II and III

Which of the following is an advantage of annual renewable term life insurance? Term life insurance accumulates cash value. Initially, term life insurance has a lower premium than whole life insurance. Initially, term life insurance provides more insurance protection per premium dollar than whole life insurance. A) II and III B) I, II, and III C) III only D) I and II Explanation Term life insurance does not accumulate cash value. Term life insurance provides only temporary protection for a given number of years. In the long run, term life insurance provides less insurance protection per premium dollar than whole life insurance.

A) II and III

Nonqualified plans provide the same tax advantages as a qualified plan. are less flexible than qualified plans. may defer taxes on compensation to employees. are usually used to supplement qualified retirement benefits to key employees. A) III and IV B) II, III, and IV C) III only D) I and IV Explanation Nonqualified plans do not provide the same tax advantages as qualified plans and are more flexible than qualified plans. Because compensation to employees is deferred, the income taxes on that compensation are deferred as well. Nonqualified plans are generally provided only to key employees.

A) III and IV

Several years ago, Diego purchased a $400,000 whole life insurance policy on his life. He has paid cumulative premiums over the years of $20,000 and has accumulated a cash value of $25,000. This year, he was diagnosed with a rare liver disease, and, as a result, his life expectancy is only six months. Because of his large medical costs, he is considering selling his policy to a viatical settlement company. The company has offered him $250,000 for the policy. He would also like to explore other ways to generate cash from the policy. Which of the following statements regarding Diego's situation are CORRECT? If Diego sells his policy to the viatical settlement company, he will be taxed on any gain from the sale if he dies more than two years later. If the viatical company collects the death benefit as a result of Diego's death, the proceeds will be tax free to the company. If Diego sold the policy to his cousin for $250,000, his cousin would be subject to ordinary income tax on a portion of the life insurance benefit when Diego dies. If Diego takes a loan from the policy, some or all of the loan will be subject to ordinary income tax if the policy is classified as a modified endowment contract (MEC). A) III and IV B) II and III C) I and II D) I, II, and IV Explanation The answer is III and IV. Because Diego is terminally ill (i.e., expected to die within two years), he will not be taxed on the proceeds received from the viatical settlement company, even if he lives longer than two years. When the viatical settlement company receives the death benefit, part of the death benefit will be taxed at ordinary income tax rates to the company. The sale of the policy to Diego's cousin would be considered a transfer for value. His cousin would be taxed on the death benefit (less any amounts paid) because the transfer-for-value rules cause the death benefit to become taxable. With a MEC, loans or distributions from the policy are taxed on a last in, first out basis, meaning that any earnings in the policy are taxed first.

A) III and IV

Which of the following statements describes how the Affordable Care Act limits the definition of pre-existing conditions? A) Limitations can no longer be applied to pre-existing conditions. B) As long as the individual has been covered by a policy previously and continuously, pre-existing conditions do not apply. C) Except for late enrollees, the limitation applicable to pre-existing conditions can be for only six months following the date of enrollment. D) Pre-existing conditions can be defined only as conditions treated within 12 months prior to the effective date. Explanation The answer is limitations can no longer be applied to pre-existing conditions. Limitations can no longer be applied to pre-existing conditions at all under any circumstances for health insurance.

A) Limitations can no longer be applied to pre-existing conditions.

Which of the following terms in a group health insurance plan describes the limits that a person will have to pay in any calendar year for covered medical expenses? A) Maximum out of pocket (MOOP) B) Coinsurance C) Copayment D) Deductible Explanation The maximum out of pocket (MOOP) is the absolute most the insured will pay for covered expenses and includes the deductible plus the insured's share of the coinsurance.

A) Maximum out of pocket (MOOP)

Which of the following statements describing insurance terms are CORRECT? A misrepresentation is carelessness or indifference on the part of an individual pertaining to whether a loss occurs and/or the size of a loss if one does occur. A morale hazard is a false and material statement made by an applicant for insurance, providing a basis for the insurer to make the contract voidable. A) Neither I nor II B) I only C) Both I and II D) II only Explanation The answer is neither I nor II. Neither statement I nor II is correct. Statement I is incorrect because it describes a morale hazard. Statement II is incorrect because it describes a misrepresentation.

A) Neither I nor II

Which of the following statements regarding long-term care expenses is CORRECT? Medicare is one of the best ways to pay for long-term care expenses because the benefits can be used for up to five years. Medicaid planning (transferring assets in order to qualify for Medicaid) is an important tool that planners should use to help their clients meet their long-term care goals. A) Neither I nor II B) I only C) Both I and II D) II only Explanation The answer is neither I nor II. Medicare is not a good option to pay for long-term care because it is limited, restrictive, and will only pay for up to 100 days of skilled nursing care. Financial advisers and planners who assist senior citizens in Medicaid planning can be convicted of a misdemeanor if they knowingly advise a client to make a transfer that results in the imposition of a look-back period.

A) Neither I nor II

Which of the following types of loss is NOT covered in a standard personal auto policy (PAP)? A) Personal property damaged in an auto accident B) Legal liability C) Injury to the insured and her family D) Damage to the vehicle Explanation The answer is personal property damaged in an auto accident. Damage to personal property in an auto accident is not covered in a standard PAP. This type of loss would normally be covered in a homeowners policy.

A) Personal property damaged in an auto accident

Your client, Carol, has brought her declaration page and the following information to you. The market value of the residence is $468,000. Her builder advised her that to rebuild her house today, it would cost $450,000. According to the declarations page of the policy, it is HO-5 insured for $425,000 with a $1,000 deductible. The value of the land is $55,000. Which of the following statements is true regarding a total loss of Carol's house? A) She will have an out-of-pocket cost of $26,000 if she rebuids the house exactly the way it is today. B) Because it is an HO-5 policy, it covers replacement, so she will have no out-of-pocket costs other than her deductible. C) The value of the land deducted from the residence's market value is less than the amount insured ($425,000); therefore, Carol's only out-of-pocket cost will be her deductible. D) She will have an out-of-pocket cost of $44,000 if she has the house rebuilt exactly the way it is today. Explanation Just because a policy is one of replacement, it does not mean it will pay more than the insured amount. In this case, the maximum the company will pay is $425,000. The true statement is that her loss would be $26,000. ($425,000 coverage less $450,000 cost to rebuild, less the $1,000 deductible: $425,000 − $450,000 −$1,000 = −$26,000. The market price and land price are irrelevant. What is important is cost to rebuild minus insured amount.

A) She will have an out-of-pocket cost of $26,000 if she rebuids the house exactly the way it is today.

Betty is a highly compensated employee at Sound Trends, a store that sells audio and video items. The company provides its employees with various benefits, including long-term disability (LTD) income coverage, at no cost to the employee. The company provides LTD coverage by purchasing an insurance policy from a nationally known company. The company pays the premiums for all covered employees. Assume that Betty is disabled for six months and receives $3,000 in LTD benefits from the plan. Which of the following explains the income tax implications of this long-term disability coverage? A) The benefits are taxable to Betty because the cost of the coverage was paid by her employer. B) The premiums are not deductible by the employer because the policy is underwritten by a third-party insurance company. C) The benefits received are tax free to Betty because they are considered a tax-free employee benefit. D) The premiums paid by Sound Trends are taxable income to Betty. Explanation The answer is the benefits are taxable to Betty because the cost of the coverage was paid by her employer. If the employer pays the premiums, benefits are taxable to the employee. If the employee pays the premiums, benefits are tax free to the employee.

A) The benefits are taxable to Betty because the cost of the coverage was paid by her employer.

The Kennedys had a kitchen fire causing extensive damage to their dwelling. The cost of repairing the damage is $40,000. At the time of the loss, the limit on the dwelling under the Kennedys' policy was $200,000, but the adjuster estimates the replacement cost to be $250,000. What is the amount of loss the Kennedys will recover from their insurance company? A) The cost of the repairs is covered in full, minus the deductible. B) The Kennedys violated the replacement cost provision, and only the actual cash value of the loss will be paid, minus the deductible. C) The Kennedys will collect $32,000, minus the deductible. D) The Kennedys will collect $40,000 if the policy includes the replacement cost endorsement. Explanation Because the Kennedys complied with the 80% coinsurance requirement for partial losses, they will collect the full cost of the repairs, less the deductible.

A) The cost of the repairs is covered in full, minus the deductible.

Devin and Lana are meeting with you next week and will bring information regarding Devin's group disability income insurance plan. What definition of disability would provide the most favorable protection for Devin? A) The inability of the insured to engage in his own occupation B) The inability of the insured to engage in any occupation C) The inability of the insured to engage in his own occupation and not working in any gainful employment D) The inability of the insured to engage in any reasonable occupation for which he is or might easily become qualified Explanation The most favorable protection for Devin would be his inability to engage in his own occupation. The own occupation definition of disability provides the broadest protection for insureds, in that they must not be able to perform the major duties of their own occupations.

A) The inability of the insured to engage in his own occupation

Which of the following is the CORRECT definition of the term adverse selection? A) The tendency of persons with the highest risk of loss to also be the most likely to purchase insurance, thereby causing the insurer the greatest cost in terms of claims B) The tendency of persons with the lowest risk of loss to also be the most likely to purchase insurance, thereby causing the insurer the greatest cost in terms of claims C) The tendency of persons with the lowest risk of loss to also be the most likely to purchase insurance, thereby causing the insurer the greatest profit D) The tendency of persons with the highest risk of loss to also be the most likely to purchase insurance, thereby causing the insurer the greatest profit Explanation The answer is the tendency of persons with the highest risk of loss to also be the most likely to purchase insurance, thereby causing the insurer the greatest cost in terms of claims. Adverse selection is the tendency for those most likely to incur losses to seek insurance.

A) The tendency of persons with the highest risk of loss to also be the most likely to purchase insurance, thereby causing the insurer the greatest cost in terms of claims

Melissa was involved in an accident that left her unable to work for an extended period of time. She has a policy that will pay her a stated amount of income on a regular basis while she is off work. Melissa most likely has A) disability income insurance. B) long-term care insurance. C) hospital income insurance. D) hospital/medical expense insurance. Explanation The answer is disability income insurance. Disability income insurance is also called income replacement insurance or loss-of-time coverage.

A) disability income insurance.

Group long-term disability insurance typically provides benefits for any of the following periods except A) for six months only. B) until an employee's normal retirement age (usually 65). C) for a specified term longer than two years. D) until death. Explanation The answer is for six months only. Group long-term disability insurance typically provides coverage for a specified term longer than two years, until an employee's normal retirement age (usually 65), or until death. A short-term disability income policy provides coverage for up to two years.

A) for six months only.

Tax deferral in an unfunded plan A) may be achieved if plan assets are subject to company creditors. B) may be achieved even if an executive's compensation is subject to constructive receipt. C) depends on whether an executive's right to compensation is subject to a substantial risk of forfeiture. D) may be achieved if the deferral is agreed upon at any time prior to the compensation being paid. Explanation Tax deferral in an unfunded plan may be achieved if plan assets are subject to company creditors. Substantial risk of forfeiture conditions do not need to be specified in an unfunded plan. If constructive receipt is deemed to have taken place, the executive is taxed. The employer's promise must be merely a naked promise and must not be secured in any fashion. The deferral must be agreed upon before the compensation is earned.

A) may be achieved if plan assets are subject to company creditors.

Liability insurance is primarily concerned with the financial consequences of A) negligence. B) felony crimes. C) all torts. D) intentional torts. Explanation Liability insurance is primarily concerned with negligence. It does not cover intentional acts.

A) negligence.

Closely held corporations that would like to reward valuable employees but do not want additional shareholders may consider using A) phantom stock. B) restricted stock. C) NQSOs. D) junior class shares. Explanation The answer is phantom stock. Closely held corporations wishing to reward their highly valued employees without adding more shareholders may use phantom (or shadow) stock arrangements. Such arrangements are structured as fictional deferred compensation accounting entries, where the base value is equal to the current value of the corporation's common stock.

A) phantom stock.

A rabbi trust provides A) security for the employee against the employer's unwillingness to pay deferred compensation benefits. B) security against employer bankruptcy. C) income tax payments for employees. D) an immediate income tax deduction for employers. Explanation While the assets held in a rabbi trust are still subject to the company's general creditors, the trust does provide the executive with security against the employer's unwillingness to pay.

A) security for the employee against the employer's unwillingness to pay deferred compensation benefits.

Regarding the Kennedys' deferred variable annuity, A) the amount of growth during the accumulation stage is uncertain. B) the insurer directs the cash value investment allocation. C) the return credited to the contract is fixed. D) the contract fees will be less than if they owned a mutual fund. Explanation Because the annuity is variable, the Kennedys have the ability to invest in one or more subaccounts with varying investment styles. Because the final outcome of the investment is uncertain, the future amount of growth during the accumulation stage is uncertain as well.

A) the amount of growth during the accumulation stage is uncertain.

All of the following statements regarding the taxation of nonqualified stock options are correct except A) the stock's holding period, for purposes of determining long- or short-term capital gains treatment to the employee, begins with the grant date. B) the employer receives a deduction for the amount of the bargain element when the employee brings the amount into income. C) on the exercise date, the bargain element represents income to the employee. D) if the nonqualified stock option does not have a readily ascertainable fair market value at the date of the grant, the employee is not taxed until the date of the exercise of the option. Explanation The stock's holding period, for purposes of determining long- or short-term capital gains treatment to the employee, begins with the exercise date. All of the other statments are correct.

A) the stock's holding period, for purposes of determining long- or short-term capital gains treatment to the employee, begins with the grant date.

Which of the following is the role of the legislative branch in regulating the insurance industry within a state? A) Passing laws relative to the insurance industry B) Enforcing the laws in place relative to the insurance industry C) Creating model legislation relative to the insurance industry D) Interpreting and applying the laws in place relative to the insurance industry Explanation The answer is passing laws relative to the insurance industry. The legislative branch votes on and passes laws relative to the insurance industry. These laws are enforced by the executive branch, and disputes concerning the interpretation or application of the law is handled by the judicial branch. The National Association of Insurance Commissioners provides model legislation that may or may not be enacted by the several states.

A) Passing laws relative to the insurance industry

Which of the following statements regarding self-insurance by a business is CORRECT? A) The self-insurer must be able to competently manage the investment of the self-insurance fund. B) Self-insurance is a method of risk transfer. C) Most firms are good candidates for self-insurance. D) The organization should have enough homogeneous exposure units to make losses unpredictable. Explanation The self-insurer must be able to competently manage the investment of the self-insurance fund. Relatively few firms are good candidates for self-insurance, which is a method of risk retention. The organization that self-insures should have enough homogeneous exposure units to make losses somewhat predictable.

A) The self-insurer must be able to competently manage the investment of the self-insurance fund.

Two years ago, Marilyn, an executive vice president with ABC Bank, was granted a nonqualified stock option (NQSO) to purchase 100 shares of ABC stock at $10 per share. When the fair market value (FMV) of the stock reached $30 per share this year, she exercised the option. She is in the 24% marginal income tax bracket. If Marilyn chooses to use a cashless exercise, how many shares of ABC stock will she own after the transaction? A) 100 B) 51 C) 30 D) 49 Explanation The answer is 51. The cost to exercise the options is $1,000 (100 shares × $10 per share). Also, because the option is an NQSO, Marilyn will have to pay ordinary income taxes of $480 on the bargain element [($30 per share FMV - $10 per share exercise price) × 100 shares × 24% tax rate]. Marilyn's total cost to exercise the option is $1,480. Because this is a cashless exercise, enough shares of stock must be sold to cover Marilyn's cost of the option. Therefore, 49 shares of the stock must be sold ($1,480 ÷ $30 per share FMV). Marilyn will have 51 shares remaining (100 − 49).

B) 51

According to Medicaid law, how many months long is the look-back period for asset transfers to others? A) 12 months B) 60 months C) 24 months D) 36 months Explanation The answer is 60 months. Medicaid law includes a provision defining a look-back period of 60 months for assets transferred to others (usually adult children) designed to impoverish the donor to become eligible for Medicaid. If a transfer is made within this 60 month look-back period, the applicant is ineligible for Medicaid benefits for a length of time equal to the amount transferred divided by the monthly cost of nursing home care for the region.

B) 60 months

Which of the following is covered under the uninsured motorist coverage of a personal auto policy (PAP)? A hit-and-run driver. A car that is operated without liability insurance. A) II only B) Both I and II C) Neither I nor II D) I only Explanation Both hit-and-run drivers and cars operated without liability insurance are insured under uninsured motorist coverage.

B) Both I and II

Which of the following statement(s) regarding phantom stock plans is CORRECT? These plans provide for adjustments to the fictional entries to track the real appreciation of the corporation's common stock. At some event (usually retirement), the theoretical investment, valued to reflect cumulative gains and losses, is paid to the employee or the employee's designated beneficiaries. A) II only B) Both I and II C) Neither I nor II D) I only Explanation The answer is both I and II. Both statements I and II are correct.

B) Both I and II

Which of the following statements regarding the income tax doctrines that apply to nonqualified deferred compensation plans is CORRECT? If constructive receipt occurs, the executive must report the funds that are constructively received as taxable income. The economic benefit doctrine does not apply if the funds placed in a nonqualified plan remain subject to potential attachment by the employer's general creditors. A) Neither I nor II B) Both I and II C) I only D) II only Explanation The answer is both I and II. Both of these statements are correct.

B) Both I and II

Once you have analyzed and evaluated the information to identify a client's risk exposures, what is the next step in the risk management process? A) Identify risk management goals. B) Develop a risk management plan. C) Monitor the plan for needed changes. D) Gather pertinent data to determine risk exposure. Explanation The steps in the risk management process are to: 1) identify and establish risk management goals, 2) gather pertinent data to determine risk exposures, 3) analyze and evaluate the information to identify risk exposures, 4) develop a risk management plan, 5) communicate the recommendations, 6) implement the recommendations, and 7) monitor the recommendations for needed changes.

B) Develop a risk management plan

Which of the following is the period during which the owner of a life insurance policy is allowed to pay an overdue premium? A) Contestable period B) Grace period C) Reinstatement period D) Waiver of premium period Explanation The answer is grace period. The insurance remains in force during the grace period. The grace period prevents the policy from lapsing by providing the policyowner with additional time to pay an overdue premium. If the insured dies within the grace period, the company deducts the overdue premium from the death benefit payable to the beneficiary.

B) Grace period

Which of the following forms insures the dwelling and other structures on an open-perils basis? A) HO-1 B) HO-3 C) HO-4 D) HO-2 Explanation The answer is under HO-3. All direct physical losses are covered, unless specifically excluded.

B) HO-3

Which of the following are insurance company rating services? Moody's A.M. Best Standard and Poor's Fitch Ratings A) I and II B) I, II, III, and IV C) I, II, and III D) II and IV Explanation The answer is I, II, III, and IV. All of these are insurance company rating services.

B) I, II, III, and IV

The tax treatment of junior class shares of stock include which of the following? The employee is not taxed when the junior class shares are purchased and issued. The employee is not taxed when the junior class shares are converted to common shares. The employee is taxed when the junior class shares are converted to common shares and then sold. When the common shares are sold, the employee incurs ordinary income taxes based on the sales price. A) III and IV B) I, II, and III C) I and II D) I, II, III, and IV Explanation The answer is I, II, and III. The employee is neither taxed on the date the junior class shares are issued nor when the junior class shares are converted to common shares. Upon sale of the common shares, the employee is taxed on the capital gain (the difference between the sales price of the stock and the employee's basis in the stock). The employee's basis is the amount paid for the junior class shares.

B) I, II, and III

Which of the following are important when selecting an insurance producer? Experience Training Education Production awards earned A) I, II, III, and IV B) I, II, and III C) IV only D) II and III Explanation The answer is I, II, and III. Options I, II, and III, along with competence, inclination to service, specialization, and a good reputation, are important when selecting an insurance producer. The production awards earned are an indication of sales ability, but not necessarily what should be sought in an insurance producer.

B) I, II, and III

Which of the following homeowners coverage amounts are specified as a percentage of the Coverage A amount? Coverage B Coverage C Coverage D Flood insurance rider A) I, II, III, and IV B) I, II, and III C) I and IV D) II and III Explanation The answer is I, II, and III. If a homeowner has flood insurance, it is under a separate policy and not stated as a percentage of Coverage A in the homeowners insurance policy.

B) I, II, and III

Which of the following services are covered by Medicare Part B? Physicians services Outpatient hospital services Diagnostic tests and x-rays Hospice care A) I and II B) I, II, and III C) IV only D) I, II, III, and IV Explanation The answer is I, II, and III. Hospice care is covered under Part A, not Part B.

B) I, II, and III

Which of the following is a principle of the workers' compensation laws? In exchange for benefits, an employee gives up the right to sue the employer. The costs for workers' compensation benefits are funded through payroll taxes, and the employee is expected to contribute. The injured employee is not required to prove negligence on the part of the employer. Many states permit, or require, individuals who have domestic help to obtain workers' compensation insurance. A) I, II, III, and IV B) I, III, and IV C) I only D) II and III Explanation The answer is I, III, and IV. Workers' compensation insurance is paid through premiums paid solely by the employer.

B) I, III, and IV

Which of the following characteristics of life insurance contracts create favorable tax treatment? Income taxes on investment gains are taxable in the year of the gain. Death benefits paid to a beneficiary are not usually taxable as income. The earnings on the cash value are not taxed during the accumulation period. A) I and III B) II and III C) II only D) I, II, and III Explanation The answer is II and III. Income taxes on investment gains are tax-deferred.

B) II and III

Question #3 of 30 Question ID: 1357909 Which of the following statements regarding Medigap insurance is CORRECT? Medigap policies may duplicate some benefits provided by Medicare. All standard Medigap policies must include at least some of cost of the first three pints of blood. A) I only B) II only C) Neither I nor II D) Both I and II Explanation The answer is II only. All standard Medigap plans must include at least some of the cost of the first three pints of blood. Medigap policies may not duplicate benefits provided by Medicare.

B) II only

Which of the following are characteristics of phantom stock plans? Company stock is issued to the employee as a reward for meeting performance goals. Each year, the employee becomes vested in shares according to a written agreement. The plan is a way to reward employees without giving them actual ownership in the company. Upon retirement or some other event, the employee receives the vested value of his account in cash. A) I and II B) II, III, and IV C) III and IV D) I, II, and III Explanation Under a phantom stock plan, no stock is actually issued to the participant. The plan is a means for companies to give employees a bonus tied to performance and reward employees without giving them actual ownership in the company. The employee becomes vested in shares each year according to a written agreement. Upon retirement or termination, the employee receives the vested value of his account in cash either as a lump sum or in installments.

B) II, III, and IV

Which of the following policies does NOT include liability coverage? A) Homeowners policies B) Inland marine policies C) Businessowners policies D) Automobile policies Explanation The answer is inland marine policies. Auto, home, businessowners policy, and liability umbrella policies all include liability coverage, while an inland marine policy does not. Inland marine policies only provide coverage on the listed property.

B) Inland marine policies

Your client, Margarita, had an upstairs window of her home broken by hail. She reported the claim, and for the two weeks following the storm, the weather had been quite nice. She rarely went into the room with the broken window and did nothing to cover the opening. During the third week following the incident, there was a severe rainstorm, and water came into the opening, damaging the floor of the room with the broken window and the ceiling of the room beneath it. On what grounds could the insurance company refuse to pay for the damaged floor and ceiling? A) Margarita failed to provide assistance and to cooperate with the insurance company. B) Margarita failed to protect the property when she had adequate time to do so. C) Margarita is estopped from collecting for damages. D) Margarita waived her right to collect for damages. Explanation The insurance company will pay for the broken window and any other damage from the hail storm, but could deny the claim for the damaged floor and ceiling since Margarita failed to protect the property when she had ample time to do so.

B) Margarita failed to protect the property when she had adequate time to do so.

Which of the following government programs may pay for Harry's nursing home expenses, assuming he is indigent and requires only custodial care? A) Medicare Part B B) Medicaid C) Medicare Part D D) Medicare Part A Explanation The answer is Medicaid. Medicaid may pay for custodial nursing home care if the patient is indigent. Medicare Part A pays some nursing home expenses if the patient needs skilled nursing care, but does not pay for custodial care. Medicare Part B does not cover nursing home expenses, and Medicare Part D covers prescription drugs.

B) Medicaid

Augusto is not paying attention when driving to work and runs a stop sign, narrowly missing Regena. Although frightened by the event, Regena continues driving and uses her cell phone to call her spouse to tell him what just happened. While dialing the phone, she crashes into a telephone pole. Would she be able to hold Augusto liable for the damage to her car? A) Yes, because Augusto breached his duty to avoid damaging Regena's car. B) No, because Augusto's actions were not the proximate cause of Regena's loss. C) No, because Augusto did not owe Regena a duty to drive his car in a safe manner. D) Yes, because there were actual damages as a result of Augusto's actions. Explanation To be liable for an unintentional tort, four elements of negligence must exist. First, a duty is owed, and second, that duty is breached. Then, there must be actual damages and, finally, the breach must be the proximate cause for the damage. In this case, Augusto had a duty to drive his car in a safe manner, and he and breached that duty when he ran the stop sign. While Regena's car did, in fact, have actual damages, the proximate cause of the damage was her trying to use her cell phone while driving. Proximate cause is an uninterrupted sequence of events that brings about damage, and Regena using her cell phone interrupted that sequence. Had she swerved to avoid Augusto and hit a telephone pole, then the proximate cause would have been Augusto's actions and he could be found negligent.

B) No, because Augusto's actions were not the proximate cause of Regena's loss.

Which of the following is the role of the executive branch in regulating the insurance industry within a state? A) To interpret and apply the laws in place relative to the insurance industry B) To enforce the laws in place relative to the insurance industry C) To create model legislation relative to the insurance industry D) To pass laws relative to the insurance industry Explanation The answer is to enforce the laws in place relative to the insurance industry. The legislative branch votes on and passes laws relative to the insurance industry. These laws are enforced by the executive branch, and disputes concerning the interpretation or application of the law is handled by the judicial branch. The National Association of Insurance Commissioners provides model legislation that may or may not be enacted by the several states.

B) To enforce the laws in place relative to the insurance industry

A cafeteria plan consisting of various tax-free benefits that are funded through salary reductions elected by employees each year is called A) a Section 162 plan. B) a flexible spending account (FSA). C) a VEBA. D) a split-dollar benefit plan. Explanation The answer is a flexible spending account (FSA). An FSA is a cafeteria plan consisting of various tax-free benefits that are funded through salary reductions elected by employees each year.

B) a flexible spending account (FSA).

One characteristic of short-term disability coverage used in an employee benefit plan is that it normally provides A) for the same elimination period for disability due to either sickness or accident. B) a weekly benefit for up to a maximum of 52 weeks. C) for benefits to be paid only after a 60-day elimination period. D) a benefit of up to 50% of salary. Explanation The answer is a weekly benefit for up to a maximum of 52 weeks.

B) a weekly benefit for up to a maximum of 52 weeks.

All of the following are exclusions from Coverage C: Personal Property of a homeowners policy except A) property of roomers or boarders. B) jewelry. C) credit cards. D) animals, birds, and fish. Explanation Jewelry is covered under Coverage C; however, coverage is typically limited to a maximum dollar amount of $1,500. The addition of an endorsement can increase the coverage limit for jewelry and other personal property items.

B) jewelry.

Joe was involved in an accident at the plant where he works, and as a result, lost his arm. Under the workers' compensation system, this type of injury is considered an example of A) partial temporary disability. B) partial permanent disability. C) total temporary disability. D) total permanent disability. Explanation Losing an arm is an example of a partial permanent disability.

B) partial permanent disability.

Mary, 54, is an executive with ABC Corporation, which provides a nonqualified deferred compensation (NQDC) plan. If Mary has unrestricted access to the assets in her NQDC plan, A) the assets are subject to a substantial risk of forfeiture, and she is not currently taxed. B) she has constructive receipt of the funds. C) the assets are subject to the claims of the employer's general creditors. D) she will incur a 25% penalty. Explanation If Mary has unrestricted access to the funds, she is considered to have constructive receipt of the income and will be taxed in the current year.

B) she has constructive receipt of the funds.

Your client wants to provide his spouse with $6,000 of additional income adjusted for inflation, paid at the beginning of each month, for 30 years after his death. He wants to use up the entire amount of principal and interest during that period. He expects 3% inflation and 7% net interest on savings and wants the income to be adjusted annually for inflation. Assuming he dies today, how much additional life insurance is required to fund this need? A) $1,274,634 B) $1,256,767 C) $1,278,760 D) $1,260,957 Explanation The answer is $1,278,760. The solution to this can be done in a single step. We're looking for the amount it would take today to fund an increasing monthly benefit for 30 years. With the calculator set for BEG mode (SHIFT, MAR), the keystrokes would be 30 × 12 = 360, N; 1.07 ÷ 1.03 = 1.0388 - 1 = .03833 × 100 = 3.8835 ÷ 12 = .3236, I/YR; 6,000, PMT, and solve for PV = 1,278,760.

C) $1,278,760

What is the amount of coverage the Kennedys have on their personal property under their homeowners insurance policy? A) $20,000 B) $200,000 C) $100,000 D) $50,000 Explanation The answer is $100,000. The Kennedys have $200,000 of coverage on their home. Under a homeowners policy, the limit on personal property (Coverage C) is typically 50% (50% × $200,000 = $100,000) of the amount of coverage on the dwelling.

C) $100,000

The Smith family has a major medical policy that provides for the following: $250 per person deductible (3-person maximum) $5,000 maximum out-of-pocket 80/20 coinsurance During their vacation, the Smiths were involved in an automobile accident in which all four family members were hurt. Each person incurred medical expenses of $7,500. How much will the insurance company pay in reimbursing the Smiths for their injuries? A) $25,500 B) $28,000 C) $25,000 D) $28,250 Explanation The answer is $25,000. The Smith family will have to pay $5,000. Recall, the deductible is limited to three people. The insurance company will pay the remainder of the claim, or $25,000.

C) $25,000

Jan pays for her group disability income policy via a flexible spending account. What amount of her benefit would be taxable as ordinary income if she became disabled? A) 0% B) 50% C) 100% D) The amount in excess of 2% of adjusted gross income Explanation The answer is 100%. Group disability benefits received via a plan funded with pretax dollars are fully taxable to the employee.

C) 100%

Which of the following statements best describes a morale hazard? A) A false and material statement made by an applicant for insurance, providing a basis for the insurer to void the contract B) An unintentional tort in the form of an action or omission that leads to the injury of another party C) A condition of carelessness or indifference on the part of an individual as to whether a loss occurs and/or the size of a loss if one does occur D) An act or condition that increases the likelihood of the occurrence of a peril and/or increases the severity of a loss if a peril does occur Explanation A morale hazard is a condition of carelessness or indifference as to whether a loss occurs and/or the size of a loss if one does occur. A homeowner's failure to lock the house doors at night is considered a morale hazard because it increases the probability of theft or loss.

C) A condition of carelessness or indifference on the part of an individual as to whether a loss occurs and/or the size of a loss if one does occur

Under a standard personal auto policy (PAP), damages are based on which of the following criteria? A) Replacement cost B) Appraisal value C) Actual cash value D) Stated value Explanation Under a standard PAP, coverage for damages is paid on an actual cash value basis, which is the replacement cost minus depreciation. There is no replacement cost coverage under a standard PAP.

C) Actual cash value

As part of a life insurance needs analysis, the Kennedys should be concerned about a fund for which of the following? A) Dependency care income B) Income for the surviving spouse C) All of these D) Education of any dependents Explanation The answer is all of these. The financial needs analysis method, also known as the life insurance needs method, analyzes all recurring expenses of the dependent survivors and any unusual expenses that may result from the death of the insured.

C) All of these

Which of the following definitions best describes a single premium immediate annuity (SPIA)? A) An annuity specifying that, if the annuitant dies before receiving total benefit payments equal to the purchase price of the annuity, the difference will be refunded in the form of continuing benefit payments B) An annuity whose benefit payments continue for the lifetimes of two or more beneficiaries C) An annuity whose benefit payments begin one payment interval after the date of purchase D) A life annuity that provides a guaranteed minimum number of benefit payments whether the annuitant lives or dies Explanation The answer is an annuity whose benefit payments begin one payment interval after the date of purchase. A SPIA is an annuity whose benefit payments begin one payment interval after the date of purchase.

C) An annuity whose benefit payments begin one payment interval after the date of purchase

Select the common characteristics of preferred provider organizations. Participating providers are paid on a fee-for-service basis as their services are used. Covered individuals have financial incentives to receive treatment within the preferred provider network. A) I only B) II only C) Both I and II D) Neither I nor II Explanation The answer is both I and II. Both statements I and II are correct.

C) Both I and II

Which of the following are features of managed care plans? Copayments Primary care physicians (PCPs) Gatekeepers Deductibles A) II and III B) II, III, and IV C) I and IV D) I, II, III, and IV Explanation Copayments, PCPs, gatekeepers, and deductibles are all features of managed care plans.

D) I, II, III, and IV

Which of the following organizations provides a broad range of health services to a group of subscribers for a fixed monthly fee and involves a capitation fee? A) Third-party administrator B) Commercial insurance company C) HMO D) Private health insurer Explanation HMOs provide health care on a prepayment basis. Blue Cross and Blue Shield organizations provide health insurance and handle both the administrative and claims processes (third party adminstrator). Self-insurance is another method of insurance in which a company hires a third-party administrator to perform all the required plan functions. Private health insurance is provided by commercial insurance companies, independent plans, and Blue Cross and Blue Shield organizations.

C) HMO

Which of the following statements, with respect to making a Section 83(b) election on restricted stock, is CORRECT? An employee who receives restricted stock may elect under Section 83(b) to recognize the income immediately rather than wait until the substantial risk of forfeiture expires. The Section 83(b) election must be made within one year of receiving the restricted stock. If the election is made, the employee immediately includes as long-term capital gain the fair market value of the stock at time of receipt, less any amount paid for the stock. If the employee makes the election and then forfeits the stock, the employee is not allowed a deduction or refund of tax on previously reported income. A) I, III, and IV B) I only C) I and IV D) II, III, and IV Explanation The answer is I and IV. The Section 83(b) election must be made within 30 days of receiving the restricted stock. The employee recognizes ordinary income and not long-term capital gain when the election is made.

C) I and IV

For an executive to retain the tax benefits of deferred compensation, she must not have constructive receipt of the funds. the funds must not be set aside in a specific account for the executive. she must have substantial risk of forfeiture. the employer must make an unsecured promise to pay the compensation. A) III and IV B) I and II C) I, II, III, and IV D) I, II, and III Explanation All of these statements must be true for an executive to retain the tax benefits of deferred compensation.

C) I, II, III, and IV

Most automobile insurance companies use a classification system to determine premiums. Which of the following factors is traditionally used in a classification system? The location of the auto The age of the driver The gender of the driver The driving record of the driver A) I only B) II and IV C) I, II, III, and IV D) I, II, and III Explanation All of these factors are used in classification systems to determine automobile insurance premiums.

C) I, II, III, and IV

Numerous requirements must be met under the Internal Revenue Code (IRC) for a tax-favored incentive stock option (ISO). These include which of the following? The written plan must be approved by the stockholders of the corporation. The option's exercise date cannot exceed 10 years from the date of the grant. There is an annual limit of $100,000 on the value of the ISOs exercised during any one year to any employee; the stock is valued on the grant date. The exercise price cannot be less than the market price of the stock on the date of the grant. A) III and IV B) I, II, and III C) I, II, III, and IV D) I and II Explanation All of these are IRC requirements for ISOs.

C) I, II, III, and IV

Raul is in the process of purchasing homeowners insurance for his new residence. Which of the following factors may affect Raul's premium? The policy form and coverage provided The location of the property The amount of the deductible The level of fire protection provided by the city or municipality A) II and IV B) I only C) I, II, III, and IV D) I, II, and III Explanation The answer is I, II, III, and IV. All of these are factors used in classifying and pricing a homeowners policy. Other factors include the construction type of the residence and the amount of insurance requested by the applicant.

C) I, II, III, and IV

Which of the following are features of the Medicaid program? Provides health care benefits for the indigent and impoverished Administered by the states with financial assistance from the federal government Includes a provision defining a look-back period of 60 months for assets transferred to others designed to impoverish the donor to become eligible for Medicaid Administered by the federal government A) II and III B) I and IV C) I, II, and III D) I and II Explanation The answer is I, II, and III. Medicaid provides health care for low-income individuals. The program was established by the federal government, which also set up the regulations and minimum standards for eligibility. However, the Medicaid program is administered by each individual state. The states remit payments to eligible participants and are partially reimbursed by the federal government.

C) I, II, and III

Which of the following statements regarding variable universal life insurance is CORRECT? This policy contains investment options and no minimum guaranteed rate of return. Planners must have state variable insurance and securities licenses to sell variable universal life insurance. Cash values can decline to zero, causing the policy to lapse unless additional premium payments are made. Variable universal life insurance policies are suited for individuals with lower risk tolerances and investment experience. A) I and IV B) II and III C) I, II, and III D) I, II, III and IV Explanation The answer is I, II, and III. Variable universal life insurance policies are suited for individuals with higher, not lower, risk tolerances and investment experience.

C) I, II, and III

Whole life insurance nonforfeiture options allow a policyowner to surrender a whole life insurance policy and receive the net cash value (cash value less any outstanding policy loans). stop paying premiums on a whole life insurance policy and exchange the net cash value for a reduced paid-up single premium permanent life insurance policy. stop paying premiums on a whole life insurance policy and use the net cash value as a single premium to purchase a paid-up term life insurance policy with a face amount equal to the face amount of the original policy for a specified period. A) I and II B) II and III C) I, II, and III D) III only Explanation The answer is I, II, and III. There are three nonforfeiture options available when surrendering or discontinuing premium payments on a whole life insurance policy. First, the policyowner can surrender the policy in return for receiving the cash surrender value of the policy. Second, the policyholder leaves the cash value with the company and receives a smaller amount of fully paid-up insurance. With the third option, a policyholder leaves the cash value with the insurance company in exchange for retaining the full amount of the original policy's death benefit, but as a term insurance policy for a guaranteed period.

C) I, II, and III

Which of the following statements regarding stock appreciation rights (SARs) are CORRECT? The executive to whom they are granted does not need to have cash available to exercise the stock appreciation rights (SARs). They are similar to phantom stock, except that with SARs, the employee can choose when to exercise the right to share in the appreciation of the closely held company's stock. Like phantom stock, they require cash to create and dilute ownership. They are used heavily by closely held businesses. A) I and III B) II and IV C) I, II, and IV D) II, III, and IV Explanation Cash does not dilute company ownership. A primary feature of SARs is that dilution of ownership is avoided.

C) I, II, and IV

Which of the following policy sections do homeowners and auto policies share? Liability coverage Comprehensive coverage Medical payments coverage Duties after a loss A) I, II, III, and IV B) I only C) I, III, and IV D) I and III Explanation With the exception of comprehensive coverage (auto policies only), all of these are sections are shared by homeowners and auto policies.

C) I, III, and IV

On May 20 of this year, Magnum Corporation grants Devin incentive stock options (ISOs) enabling him to purchase 1,000 shares of Magnum stock for $20 per share. At the time of the grant, the stock was selling for $20 per share. On November 23 of this year, when the stock was selling for $25 per share, Devin exercised all of the ISOs. Which of the following statements regarding the income tax treatment of these is CORRECT? Devin must recognize ordinary taxable income of $5,000 this year. Devin recognizes no ordinary taxable income this year. Devin must recognize the $5,000 as a positive alternative minimum tax (AMT) adjustment item this year. A) II only B) I and III C) II and III D) III only Explanation Because these stock options are ISOs, Devin does not recognize any regular income at the time of exercise. However, he must recognize the bargain element ($5 per share × 1,000 shares) as a positive AMT adjustment item this year.

C) II and III

Which of the following statements regarding group long-term care (LTC) insurance is CORRECT? Generally, group LTC policies are more expensive than individual LTC policies. Employers may offer group long-term care insurance as an employee benefit. After an employee's termination, LTC coverage may be continued on a direct pay basis. A) I, II, and III B) III only C) II and III D) II only Explanation The answer is II and III. Generally, group LTC policies are less expensive that individual LTC policies.

C) II and III

A supplemental employee retirement plan (SERP) is a qualified deferred compensation plan. often referred to as a salary continuation plan. a method of providing additional retirement income to executives. a plan that complements existing retirement plans to bring executive benefits up to desired levels. A) III and IV B) I, II, and III C) II, III, and IV D) I and II Explanation The answer is II, III, and IV. A SERP is a nonqualified deferred compensation plan and is often referred to as a salary continuation plan because the executive is not required to decrease his current salary. In addition, a SERP is a method of providing additional retirement income to executives over and above that which is provided by qualified plans, and it complements existing retirement plans to bring executive benefits up to desired levels.

C) II, III, and IV

In which of the following circumstances is income considered to be constructively received by an employee? The employee's receipt of the income is subject to substantial limitations or restrictions. The income is credited to the employee's account. The income is set apart for the employee. The income is made available so the employee may draw on it anytime, or could have drawn on it during the tax year if notice of intent to withdraw had been given. A) II and IV B) I and IV C) II, III, and IV D) II and III Explanation Income is not constructively received if the employee's control of its receipt is subject to substantial limitations or restrictions.

C) II, III, and IV

Which of the following statements regarding business overhead expense (BOE) insurance is CORRECT? Benefits are received tax free by the business. BOE policies will cover the owner for his salary during disability. BOE insurance is designed to cover the expenses that are usual and necessary in the operation of the business, should the owner become disabled. A) I and III B) II and III C) III only D) I, II, and III Explanation The answer is III only. Because policy premiums are a deductible business expense to the company, the benefits payable are taxable income to the business. Most BOE policies will cover the expenses that are usual and necessary in the operation of a business, with the exception of the owner's salary.

C) III only

Which of the following is true for property coverage in a commercial package policy (CPP)? A CPP provides broader coverage but is a bit more expensive than a collection of monoline forms. No customization is allowed, as all potential risks are covered in the standard CPP. The standard CPP includes building, contents, and commercial automobile coverage. The CPP is designed for larger businesses. A) I, II, and III B) III and IV C) IV only D) I, II, III, and IV Explanation The answer is IV only. A CPP is a standard package of what used to be monoline forms offered at a discount. The standard CPP includes coverage for buildings, contents, and liability coverage. Additional coverage is available for things like commercial autos, glass, and specific, unique-to-the-business causes of loss. Thus, a CPP is customizable so it can accommodate many different businesses. The CPP is designed for larger businesses, while the business owner policy is designed for smaller businesses.

C) IV only

Debbie recently lost one of her diamond earrings, and she filed an insurance claim. Which of the following is NOT an option for the insurer? A) Replace both earrings B) Pay the difference between the actual cash value of the earrings before and after the loss C) Require Debbie to surrender the remaining earring before receiving a check D) Replace the lost earring Explanation The answer is require Debbie to surrender the remaining earring before receiving a check. Options available to the insurer to settle a claim like this include replacing both earrings, replacing just the lost earring, or simpy issuing a check for an amount equal to the value of the lost earring. The insurance company can ask but cannot require Debbie to surrender the other earring. It can reduce the settlement amount by the salvage value of the single earring if she does not surrender it.

C) Require Debbie to surrender the remaining earring before receiving a check

Larry was offered two disability policies: One was noncancelable, and the other was guaranteed renewable. Which of the following is the most likely reason he chose the noncancelable policy? A) The renewal is for a stated period only. B) Certain conditions will be excepted under the renewed contract. C) The premium is guaranteed at renewal. D) The premium may be adjusted for an entire class. Explanation Premiums on guaranteed renewable policies, while initially lower, may increase with each renewal. While initially more expensive, the noncancelable policy's premiums never increase, and so in the long run, it may end up being less expensive.

C) The premium is guaranteed at renewal.

Which of these statements is true regarding most group life insurance plans sponsored by an employer? A) Including church and government employers, they are subject to the ERISA plan termination insurance provisions. B) They are subject to the participation and vesting requirements of the Employee Retirement Income Security Act of 1974 (ERISA). C) They are required to meet ERISA's reporting, disclosure, and fiduciary responsibility provisions. D) They are considered nonqualified plans. Explanation Generally, welfare benefit plans are required to meet ERISA's reporting, disclosure, and fiduciary responsibility provisions but are exempt from the participation, vesting, funding, and plan termination provisions. (Church, government, and unfunded excess benefit plans are generally exempt from ERISA.)

C) They are required to meet ERISA's reporting, disclosure, and fiduciary responsibility provisions.

Which of the following statements regarding typical personal liability umbrella policies is NOT true? A) They generally have liability limits of at least $1 million. B) There are no standard policies. C) They commonly require a base policy with a minimum of $1 million of liability coverage. D) The individual seeking the coverage is usually required to have increased levels of homeowners and automobile insurance liability. Explanation Umbrella policies normally have liability limits of $1 million or more. Base policies normally include a minimum of $300,000 of liability coverage.

C) They commonly require a base policy with a minimum of $1 million of liability coverage.

A periodic annuity payment that is guaranteed to pay a set amount is a feature of A) a variable annuity. B) a deferred variable annuity. C) a fixed annuity. D) a deferred annuity. Explanation The answer is a fixed annuity. To guarantee fixed payments to the annuitant, the insurer invests the premiums during the accumulation period in bonds, mortgages, and other fixed-income securities with a guaranteed return.

C) a fixed annuity.

Errors and omissions (E&O) insurance provides protection A) for individuals whose liability could result in bodily harm. B) when a business transports goods domestically. C) against the deficient acts of a professional who handles money. D) for businessowners in the event of disability. Explanation The answer is against the deficient acts of a professional who handles money. E&O insurance provides protection against the deficient acts of a professional who handles money. Malpractice insurance is generally used where the deficient conduct of the insured may result in bodily harm (e.g., a physician, surgeon, or dentist). Inland marine insurance covers businesses that transport goods domestically. Business overhead expense insurance provides coverage for businessowners in the event of disability.

C) against the deficient acts of a professional who handles money.

The economic benefit doctrine (Section 83) requires A) future consulting fees to be paid after retirement to be currently included as employee income. B) the present value of a not-to-compete restriction be currently included as employee income. C) restricted stock to be included as employee income if there is no longer a substantial risk of forfeiture. D) that stock options are deductible by the employer in the year granted. Explanation The answer is restricted stock to be included as employee income if there is no longer a substantial risk of forfeiture. Section 83 requires the employee to include as income any property that the employee has the right to enjoy, or the employee's right to the property is no longer subject to a substantial risk of forfeiture. Consulting services after retirement are subject to risk of forfeiture.

C) restricted stock to be included as employee income if there is no longer a substantial risk of forfeiture.

ADM Inc., a closely held business, wishes to provide Jackie, a key employee, with a plan that gives her a choice of when to exercise the right to share in the appreciation of the company's stock. Therefore, the company should consider using A) restricted stock. B) phantom stock. C) stock appreciation rights. D) junior class shares. Explanation The answer is stock appreciation rights. Stock appreciation rights (SARs) are similar to phantom stock plans, except SARs give the employee/executive a choice of when to exercise the right to share in the appreciation of the closely held company's stock.

C) stock appreciation rights.

The right of an insurer to recover damages from a third party for a loss covered by insurance is A) insurable interest. B) indemnity. C) subrogation. D) coinsurance. Explanation Subrogation is the right of an insurance company that has paid for a loss to recover its payments if it is determined that a different insurance company or person is responsible for the loss and is required to pay for it. This prevents the insured from collecting twice for the same loss. Coinsurance may be a splitting of costs, or it may refer to a minimum percentage of insurance that is required to avoid being penalized for inadequate insurance when there are partial losses. Insurable interest exists when the interested party will suffer a financial loss if the insured loss occurs. Indemnity involves placing a person back in the position they would have been had the loss not occurred.

C) subrogation.

The principal difference among the various homeowners forms is A) the liability coverage under Section II. B) the extensions of coverage in each form. C) the perils insured against in Section I. D) the definition of dwelling in each form. Explanation The answer is the perils insured against in Section I. The principal difference among the various homeowners forms are the perils insured against in Section I of the policy. Specifically, some policies cover only basic perils, some cover broad perils, and others are written as open perils.

C) the perils insured against in Section I.

All of the following statements regarding Medicare Supplement (Medigap) plans are correct except A) Medigap plans are sold by private insurance companies. B) each plan offers a different combination of benefits, and the premium cost is proportional to the coverage provided. C) under HIPAA, Medigap policies must accept all applicants within the first nine months of when they qualify for Medicare, regardless of any pre-existing conditions. D) the rules governing the sale and offerings of Medigap policies vary from state to state. Explanation Under HIPAA, Medigap policies must accept all applicants within the first six (not nine) months of when they qualify for Medicare, regardless of any pre-existing conditions.

C) under HIPAA, Medigap policies must accept all applicants within the first nine months of when they qualify for Medicare, regardless of any pre-existing conditions.

Coverage C in a homeowners policy insures the personal belongings of the policyowner and any resident family members. Which of the following is the typical limit on this coverage? A) 80% of the Coverage B limit B) 50% of the Coverage B limit C) 80% of the Coverage A limit D) 50% of the Coverage A limit Explanation The limit on Coverage C coverage is typically 50% of the Coverage A limit.

D) 50% of the Coverage A limit

Miguel purchased a $100,000 annuity and, based on his life expectancy, the insurance company determines he could anticipate 20 years of payments of $750 per monthly. What part of each monthly payment is taxable? A) $750.00 B) $0 C) $416.67 D) $333.33 Explanation The answer is $333.33. Part of each $750 will be taxable, and part will be a nontaxable return of capital. The nontaxable amount is determined as follows: $750×$100,000($750×12×20)=$750×0.5556=$416.67$750×$100,000($750×12×20)=$750×0.5556=$416.67 For each $750 payment, $416.67 will be a nontaxable return of capital and $333.33 will be taxable.

D) $333.33

In 2020, Judy's employer granted her nonqualified stock options to purchase 500 shares of employer stock at $20 per share. Judy exercised the options in 2021 when the fair market value of the stock was $30. What is the amount that Judy must include as W-2 compensation income in 2021? A) $7,500 B) $0 C) $15,000 D) $5,000 Explanation Judy must report the bargain element of $5,000 [($30 FMV - $20 exercise price) × 500 shares] as W-2 compensation income in 2021.

D) $5,000

Daniele has a universal life insurance policy with the Option B death benefit. The face amount is $500,000, and the current cash value is $225,000. The beneficiary is her son, Richard. If Daniele dies today, what amount will Richard receive as a death benefit? A) $275,000 B) $225,000 C) $500,000 D) $725,000 Explanation The answer is $725,000. Under a universal life insurance policy with the Option B death benefit (also known as the increasing death benefit option), the death benefit is the face amount of the policy plus the cash value. Richard will receive $725,000 ($500,000 + $225,000).

D) $725,000

Which of the following is a dividend option? A) Variable payment B) Extended term C) Life income D) Accumulated at interest Explanation The answer is accumulated at interest. Leaving dividends with the insurance company to accumulate at interest is a dividend option. Extended term is a nonforfeiture option, while variable payment and life income are settlement options.

D) Accumulated at interest

Which of the following terms is correctly matched with its definition? A) Reformation: an equitable remedy by which the original contract entered into by the parties is deemed null from its beginning. B) Doctrine of waiver: prevents a party from asserting a right to which he would otherwise be entitled where, because of his own actions or behavior, he misled someone (even though unintentionally) who relied on the understanding created, thereby to his own detriment. C) Doctrine of estoppel: a party, by her own actions (or the actions of her agent), has voluntarily relinquished a known right. D) Adhesion: one party writes the contract, and the other party has no say in the matter—she only accepts or rejects the contract. Explanation The answer is adhension: one party writes a contract, and the other party has no say in the matter—she only accepts or rejects the contract. This is an accurate description of adhesion.

D) Adhesion: one party writes the contract, and the other party has no say in the matter—she only accepts or rejects the contract.

Which of the following is NOT a definition that would qualify an insured for individual disability income benefits? A) The loss of sight, hearing, or speech, or the loss of use of both hands, both feet, or one of each B) An illness or injury that leaves the insured unable to work in any gainful occupation C) An illness or injury that prevents the insured from performing the principal duties of their own occupation D) An illness or injury that results in an inability to perform two of the five activities of daily living Explanation An illness or injury that results in an inability to perform two of the five activities of daily living applies to long-term care insurance, not disability income insurance.

D) An illness or injury that results in an inability to perform two of the five activities of daily living

Which of the following statements regarding tax considerations of nonqualified retirement plans is CORRECT? Under IRS regulations, a nonqualified plan benefit becomes currently taxable to an executive even before the benefit is actually received, if it has been constructively received by the executive. Constructive receipt occurs if the deferred compensation is credited to an executive's account, set aside for the executive, or made immediately available to the executive. A) Neither I nor II B) II only C) I only D) Both I and II Explanation The answer is both I and II. Both statements I and II are correct. LO 5.1.2

D) Both I and II

Which of the following federal laws require employers with more than 20 employees to include in their group insurance plan a continuation of benefits provision for all eligible employees? A) ERISA B) PPACA C) TEFRA D) COBRA Explanation COBRA requires employers with more than 20 employees to include in their group insurance plan a continuation of benefits provision for all eligible employees. Coverage may be continued for 18-36 months.

D) COBRA

Which type of insurance producer sells insurance from direct writer companies? A) Brokers B) Independent agents C) Solicitors D) Captive agents Explanation The answer is captive agents. Captive agents sell for direct writers, and examples of these include State Farm, Allstate, and Farmers Insurance.

D) Captive agents

Which of the following renewal provisions within a disability income insurance policy provides that the right to renewal is guaranteed, and the insurer cannot increase premiums except on a class basis? A) Nonguaranteed renewable B) Noncancelable C) Conditionally renewable D) Guaranteed renewable Explanation The answer is guaranteed renewable. When a disability income insurance policy is guaranteed renewable, the right to renew is guaranteed. The insurer may only increase the premiums for an entire class of policyholders.

D) Guaranteed renewable

Which of the following are the perils covered under basic coverage in a homeowners policy? Smoke Theft Falling objects Earthquake A) II and III B) I, II, and III C) II, III, and IV D) I and II Explanation The answer is I and II. Basic coverage provides benefits for financial loss to the policyowner's home due to fire, lightning, windstorm, hail, riot or civil commotion, aircraft, vehicles, smoke, vandalism or malicious mischief, explosion, theft, and volcanic eruption. Loss or damage due to falling objects is covered under broad coverage. Loss from an earthquake is excluded from most homeowners

D) I and II

Which of the following statements regarding group long-term care (LTC) insurance is CORRECT? The provisions of a group LTC policy are essentially the same as individual coverage. After separation from service, the employee may continue the LTC coverage on a direct pay basis. Group LTC contracts are underwritten on an individual basis. A) I, II, and III B) I only C) II only D) I and II Explanation The answer is I and II. Only statement III is incorrect. Group LTC contracts are underwritten on a group basis as an employee benefit.

D) I and II

Elise has been working for a national dental firm as a dentist and recently left to start her own practice. Which of the following should she purchase to protect herself from business-related liability risks? Malpractice insurance Errors and omissions insurance A businessowners policy (BOP) A personal liability umbrella policy (PLUP) A) I, II, and III B) I, II, III, and IV C) II and IV D) I and III Explanation The answer is I and III. As a medical professional, Elise would need malpractice insurance rather than errors and omissions insurance. A businessowners policy is also necessary, as she will be a business owner and will have business-related liability risks typical to any other business, such as people falling on the premises. A PLUP specifically excludes business-related liability, as it is a personal policy rather than a business policy. Elise could probably benefit from a PLUP's protection, but it is not for business-related risks as the question asks.

D) I and III

Personal risk exposures that can be covered by life insurance include premature death before a debt is repaid. premature death before children's education is paid. the spouse without a retirement benefit outliving the spouse who is receiving a straight life annuity pension payout. premature death prior to funding the family's financial goals. A) II, III, and IV B) I and III C) I, II, and III D) I, II, III, and IV Explanation The answer is I, II, III, and IV. All of these are personal risk exposures that can be covered by life insurance.

D) I, II, III, and IV

Which of the following statements regarding variable universal life insurance is CORRECT? This policy contains investment options and no minimum guaranteed rate of return. Cash values can decline to zero, causing the policy to lapse unless additional premium payments are made. Planners must have state variable insurance and securities licenses to sell variable universal life insurance. Variable universal life insurance policies are suited for individuals with higher risk tolerances and investment experience. A) I and IV B) II, III, and IV C) II and III D) I, II, III, and IV Explanation All of these statements regarding variable universal life insurance policies are correct.

D) I, II, III, and IV

Under a Section 1035 exchange, which of the following policies may be exchanged on a tax-free basis? An endowment policy exchanged for another endowment policy, in which the beginning date for regular payments is no later than the original contract qualified long-term care contract, or annuity contract One annuity contract exchanged for another annuity contract A life insurance policy exchanged for another life insurance policy (on the same insured), annuity, or endowment contract An annuity contract exchanged for a life insurance policy A) I and IV B) I and II C) II and III D) I, II, and III Explanation Statement IV is not an eligible tax-free exchange under Section 1035. A taxable event occurs if an annuity is exchanged for a life insurance policy or endowment contract.

D) I, II, and III

Which of the following are the major differences between a nonqualified plan and a qualified retirement plan? Tax benefits Distributions ERISA requirements A) II and III B) I and II C) I only D) I, II, and III Explanation The answer is I, II, and III. All of these are considered major differences.

D) I, II, and III

Which of the following statements concerning term life insurance is CORRECT? The convertible feature of a term life insurance policy permits the policyowner to exchange the term contract for a contract of permanent life insurance within a specified time without evidence of insurability. If the term life insurance policy is converted, the insured's attained age may determine the premium rate. Term life insurance provides life insurance protection for a limited period only. The face amount of the policy is payable if the insured dies during the specified period, and a reduced amount is paid if the insured survives. The premium for term life insurance is initially lower because most term contracts do not cover the period of old age when death is most likely to occur and the cost of insurance is high. A) III and IV B) II only C) I and III D) I, II, and IV Explanation The answer is I, II, and IV. Statement III is incorrect because nothing is paid if the insured survives the period specified in the term life insurance policy.

D) I, II, and IV

Which of the following statements regarding the tax treatment of phantom stock are CORRECT? The employee is taxed upon receiving the grant. The employee is taxed when payment is received at retirement. The employer receives a tax deduction when payment is made to the employee. The employer receives a tax deduction when the employee receives the grant. A) I, II, III, and IV B) III only C) I and II D) II and III Explanation The employee is taxed when payment is received at retirement or termination. The employer receives a tax deduction only when the employee receives payment at retirement or termination.

D) II and III

A client recently annuitized his fixed annuity and selected a life annuity with a 15-year period certain payout option. The income payments from the annuity are currently $2,000 per month. If the client dies after receiving income payments for 10 years, which of the following statements is CORRECT? A) Income payments will stop. B) Income payments will continue to the beneficiary for five years; the amount of the payments will vary depending on the investment returns of the underlying subaccounts. C) Income payments of $2,000 per month will continue to the beneficiary for 15 years. D) Income payments of $2,000 per month will continue to the beneficiary for five years. Explanation The answer is income payments of $2,000 per month will continue to the beneficiary for five years. In a life annuity with period certain payout option, if the annuitant dies before the end of the specified period, payments continue to the beneficiary for the remaining term. In this case, payments continue for five years. Because the client's annuity is a fixed annuity, the payments are not based on the investment performance of subaccounts.

D) Income payments of $2,000 per month will continue to the beneficiary for five years.

Which of the following statements regarding health savings accounts (HSAs) is CORRECT? A) HSA balances remaining unused at the end of the calendar year are forfeited. B) Employer contributions to a health savings account are included in the employee's gross income. C) An HSA can only be established by an individual, not an employer. D) Individuals age 55 and over who are covered by a high-deductible health plan can make additional catch-up contributions to their HSAs. Explanation The answer is individuals age 55 and over who are covered by a high-deductible health plan can make additional catch-up contributions to their HSA. Unused account balances remain in the account and continue to grow on a tax-advantaged basis. Employer contributions are excluded from the employee's income. Both individuals and employers can establish HSAs. (Employers establish them for employees.)

D) Individuals age 55 and over who are covered by a high-deductible health plan can make additional catch-up contributions to their HSAs.

Which of the following regarding policy loans is NOT true? A) A policyholder generally may borrow close to the entire cash value of a policy (less some interest). B) With variable products, cash value equal to the amount borrowed is moved to a guaranteed interest rate account. C) A variable interest rate is generally used with participating policies. D) Most insurers charge interest in advance. Explanation Similar to most consumer loans, insurers generally charge interest in arrears. The other statements are true.

D) Most insurers charge interest in advance.

Barbara left her car parked on top of a hill while visiting at a friend's house. Unfortunately, she forgot to apply her emergency brake, and her car rolled down the hill, injuring two children who were playing. Which of the following doctrines may influence Barbara's liability in this situation? A) Strict liability B) Assumption of risk C) Attractive nuisance D) Negligence Explanation The answer is negligence. The doctrine of attractive nuisance is exemplified by a homeowner not fencing in a below-ground swimming pool and then children drowning in the pool. Barbara, by failing to apply her emergency brake at the top of a hill, was negligent.

D) Negligence

Which of the following includes the five elements of an insurance contract? A) Offer and acceptance, consideration, legal object, accidental loss, and legal form B) Offer and acceptance, measurable risk, legal object, legal capacity, and legal form C) Offer and acceptance, consideration, legal object, legal capacity, and catastrophic loss D) Offer and acceptance, consideration, legal object, legal capacity, and legal form Explanation The five elements of an insurable contract are offer and acceptance, consideration, legal object, legal capacity, and legal form.

D) Offer and acceptance, consideration, legal object, legal capacity, and legal form

You have a client who is interested in purchasing an individual disability income insurance policy. She is an attorney and, because her practice is doing well, she is not overly concerned about the cost of the policy. She does want the most favorable definition of disability for her profession. Which of the following definitions of disability should you recommend to your client? A) Any gainful employment B) Modified own occupation C) Any occupation D) Own occupation Explanation The answer is own occupation. Clearly, the client should purchase a disability income policy including the own occupation definition. This will be the most favorable definition and make it easier to qualify for benefits.

D) Own occupation

Which type of insurance producer handles any type of insurance that cannot be purchased using normal distribution channels within a given state? A) Captive agents B) Independent agents C) Producing general agents D) Surplus line agents Explanation Surplus line agents and brokers serve a vital role by filling gaps in states where admitted insurers are unable to provide the insurance an individual may need.

D) Surplus line agents

Which of the following provides model legislation that may or may not be enacted by several states? A) HIPAA B) The Dodd-Frank Act C) The IRS D) The National Association of Insurance Commissioners (NAIC) Explanation The answer is the National Association of Insurance Commissioners (NAIC). The NAIC does not regulate the insurance industry, but rather, provides model legislation that may or may not be enacted by several states. It is not a federal law or agency.

D) The National Association of Insurance Commissioners (NAIC) Explanation

Which of the following is the source from which workers' compensation insurance premiums are paid? A) The employee to the employer B) The employer to the employee C) The employee to the state D) The employer to the state Explanation The answer is the employer to the state. Workers' compensation insurance premiums are paid by the employer to the state.

D) The employer to the state

In which of the following situations would a personal automobile policy (PAP) NOT provide coverage under the liability section of the policy? A) The named insured uses his trailer, which is hitched to his brother's truck, to move furniture. B) A resident relative, with permission, borrows a truck to haul some sod to his house. C) The named insured borrows a van while his covered auto is being repaired. D) The named insured uses his car to regularly transport executives to the airport for a fee. Explanation The answer is the named insured uses his car to regularly transport executives to the airport for a fee. The PAP contains an exclusion for vehicles used to transport people or property for a fee.

D) The named insured uses his car to regularly transport executives to the airport for a fee.

Which of the following is a common property coverage that can be included in a commercial package policy? A) Employer's liability B) Workers' compensation C) Commercial liability D) Theft by customers Explanation The answer is theft by customers. Coverage for theft by customers is a property coverage, while the other answer choices provide liability coverage.

D) Theft by customers

Pauline, age 45, would like to purchase life insurance. She wants a policy that will provide lifetime protection, and she would like to build up a cash value. Pauline has a low risk tolerance and wants the cash value to be invested in the insurance company's general account. She wants the insurance company to assume all the investment risk for the cash value, and she wants a policy that guarantees a minimum cash value at each age. Which of the following policies would best meet Pauline's needs? A) Term life B) Variable life C) Variable universal life D) Whole (ordinary) life Explanation The whole (ordinary) life policy best meets Pauline's needs because it will provide protection for her entire life (if the premiums are paid as agreed) and a guaranteed cash value. In addition, the insurance company assumes all of the investment risk and guarantees a minimum cash value at each age.

D) Whole (ordinary) life

All of the following are covered under a personal automobile policy's other-than-collision (comprehensive) coverage except A) flood. B) earthquake. C) theft. D) colliding with a mailbox. Explanation Collision is defined as "the upset of your covered auto or its impact with another vehicle or object." Other examples of losses covered under other-than-collision coverage include vandalism, hail, and breakage of glass.

D) colliding with a mailbox.

All of the following are services provided by a long-term care insurance policy except A) custodial care. B) skilled nursing care. C) adult day care. D) home cleaning service. Explanation The answer is home cleaning service. Home cleaning services are not reimbursed or provided by a long-term care insurance policy. Other covered services include intermediate nursing care, home health care, assisted living, and hospice care.

D) home cleaning service.

The granting of a stock option is generally a _____ event because the option's exercise price is usually equal to the stock's trading price on the day of the grant. A) taxable B) qualified C) discriminatory D) nontaxable Explanation The answer is nontaxable. The granting of a stock option is generally a nontaxable event because the option's exercise price is usually equal to the stock's price on the day of the grant. The option, therefore, has no readily ascertainable value, and there is no economic benefit to the employee to tax.

D) nontaxable

The appropriate amount of life insurance coverage for a client may be affected by his survivors' needs. These needs may include all of the following except A) education funding. B) emergency fund. C) regular income. D) the client's health status. Explanation The answer is the client's health status. This question is asking about beneficiary needs. The client's health status would not affect how much life insurance the client needs. Rather, that would affect whether or not the client can obtain the coverage needed and at what price.

D) the client's health status.

Which of the following are methods of managing risk? Avoidance Retention Reduction Transfer A) I and II B) III and IV C) II, III, and IV D) I, II, III, and IV Explanation The answer is I, II, III, and IV. All of these are risk management techniques. Avoidance and reduction are methods of risk control. Retention and transfer are methods of risk financing. LO 1.2.1

D) I, II, III, and IV


संबंधित स्टडी सेट्स

13. Мова і мовлення

View Set

BUL 3310 Units 6 Chapter 13 & 15

View Set

Sociology Exam 2, Interaction, Deviance and Global Inequality Chapters-4,6,8

View Set

Lumbar Spine, Sacrum, and Coccyx Anatomy Clinical exam (Summer 2019)

View Set

Romeo and Juliet vocab act 1 and 2

View Set